3
$\begingroup$

Is it consistent in $\mathsf{ZF}$ that there is an infinite cardinal $\kappa$, cardinals $\alpha, \beta\in\kappa$ and a function $f:\kappa\to \alpha$ such that for each $x\in\alpha$ there is an injective map $i:f^{-1}(\{x\})\to \beta$?

$\endgroup$
4
  • $\begingroup$ I think in ZFC one can write $\kappa=|\Gamma_f|=|\coprod_{x\in \alpha}f^{-1}(\{x\})|\leq\alpha\beta=\mathrm{max}(\alpha,\beta)<\kappa$? Which point fails in ZF? $\endgroup$
    – Qfwfq
    Nov 10, 2018 at 9:40
  • 1
    $\begingroup$ @Qfwfq How do you prove $|\coprod_{x\in \alpha}f^{-1}(\{x\})|\leq\alpha\beta$ in $\mathrm{ZF}$ if you don't have a choice function $x \mapsto i_x \colon f^{-1}(\{x\}) \to \beta$? $\endgroup$ Nov 10, 2018 at 11:37
  • $\begingroup$ Do $\alpha,\beta,\kappa$ have to be alephs? I guess so, because you are treating them as sets? $\endgroup$
    – bof
    Nov 10, 2018 at 11:42
  • $\begingroup$ @Qfwfq You're welcome. $\endgroup$ Nov 10, 2018 at 17:29

1 Answer 1

5
$\begingroup$

Yes, this is consistent.

Consider a (transitive) model of $\mathrm{ZF}$ in which $\omega_1$ has countable cofinality. Fix a strictly increasing, cofinal sequence $(\xi_n \mid n < \omega)$ in $\omega_1$. Consider $$ f \colon \omega_1 \to \omega, x \mapsto \min \{n < \omega \mid x < \xi_n \}. $$ $f^{-1}(\{n\})$ is bounded in $\omega_1$ for all $n \in \omega$. Hence there is an injection $$ i \colon f^{-1}(\{n\}) \to \omega. $$

$\endgroup$
0

Your Answer

By clicking “Post Your Answer”, you agree to our terms of service and acknowledge you have read our privacy policy.

Not the answer you're looking for? Browse other questions tagged or ask your own question.